Question

2. Consider the set of all polynomials of the form 2 + at + bt2 where a and b are real number (a). Show by means of an exampl
0 0
Add a comment Improve this question Transcribed image text
Answer #1

Date a Let P = {2+ at + bt? I a, b E IR? Let (2+ + ++2) , (2-t + t²) E P 2 + x + t² + 2 A + t² = 4 + 2+² But is 4+2t² is not

Add a comment
Know the answer?
Add Answer to:
2. Consider the set of all polynomials of the form 2 + at + bt2 where...
Your Answer:

Post as a guest

Your Name:

What's your source?

Earn Coins

Coins can be redeemed for fabulous gifts.

Not the answer you're looking for? Ask your own homework help question. Our experts will answer your question WITHIN MINUTES for Free.
Similar Homework Help Questions
  • Math 287, ME/MQ 01 3. Consider the set W of all 2 x 2 matrices of...

    Math 287, ME/MQ 01 3. Consider the set W of all 2 x 2 matrices of the form with the standard operations of matrix addition and scalar multiplication (a) Show that this set is closed under the operation of addition, (b) Ifu,EW and c is a real munber, show that cu + v) cu + cv. (c) Display the zero vector for W.

  • Determine if the given set is a subspace of P4. Justify your answer. All polynomials of...

    Determine if the given set is a subspace of P4. Justify your answer. All polynomials of degree at most 4, with integers as coefficients. Complete each statement below. The zero vector of P4 in the set because zero an integer The set v closed under vector addition because the sum of two integers an integer The set closed under multiplication by scalars because the product of a scalar and an integer an integer Is the set a subspace of P4?...

  • Let    be the set of third degree polynomials Is    a subspace of   ? Why or why not?...

    Let    be the set of third degree polynomials Is    a subspace of   ? Why or why not? Select all correct answer choices (there may be more than one). a. is not a subspace of    because it is not closed under vector addition b. is a subspace of    because it contains the zero vector of   c. is not a subspace of    because it is not closed under scalar multiplication d. is a subspace of    because it contains only second degree polynomials e. is...

  • linear algebra do both drawing in part A and B 2. Consider the set and let...

    linear algebra do both drawing in part A and B 2. Consider the set and let addition and scalar multiplication be the standard operations on vectors in R2 (a) State a specific numerical example that shows that V is not closed under vector addition, then draw a sketch for your example, shading the region V, and drawing your example vectors and their linear combination with the parallelogram method that shows V is not closed under vector addition (b) Provide a...

  • 1. Let Q be the set of polynomials with rational coefficients. You may assume that this...

    1. Let Q be the set of polynomials with rational coefficients. You may assume that this is an abelian group under addition. Consider the function Ql] Q[x] given by p(px)) = p'(x), where we are taking the derivative. Show that is a group homomorphism. Determine the kernel of 2. Let G and H be groups. Show that (G x H)/G is isomorphic to H. Hint: consider defining a surjective homomorphism p : Gx HH with kernel G. Then apply the...

  • Determine if the set V = {at? | a € R} is a subspace of the...

    Determine if the set V = {at? | a € R} is a subspace of the vector space P2 = {ao +ajt + azt? | ao, a1, az ER}. You may assume that vector addition in P2 is given by the usual addition of polynomials and that the scalars used in scalar multiplication are real numbers. If you decide that Vis a subspace of P2, then identify the zero vector in V and explain briefly why Vis closed under vector...

  • Let H be the set of third degree polynomials Let H be the set of third...

    Let H be the set of third degree polynomials Let H be the set of third degree polynomials {ax + ax? + ax3 | DEC} Is H a subspace of P3? Why or why not? Select all correct answer choices (there may be more than one). a. H is not a subspace of P3 because it is not closed under scalar multiplication b.H is a subspace of P3 because it contains the zero vector of P3 c. H is not...

  • Question 3 Consider the set E consisting of all quadratic polynomials of the form f(x) =ax2...

    Question 3 Consider the set E consisting of all quadratic polynomials of the form f(x) =ax2 + b, where a,b ER. Let g(x) = x + 3. Find the polynomial fe e such that the distance between (f(0), f(1), f(2)) and (g(0), g(1), g(2) is minimized. What is f(1)? (Computational Check: The sum of the numerator and denominator of f(1) is 61).

  • Let H be the set of third degree polynomials H = {ax + ax? + ax3...

    Let H be the set of third degree polynomials H = {ax + ax? + ax3 | DEC} Is H a subspace of P3? Why or why not? Select all correct answer choices (there may be more than one). a. A is a subspace of P3 because it contains the zero vector of P3 | b. H is not a subspace of P3 because it does not contain the zero vector of P3 c. H is not a subspace of...

  • Let H be the set of third degree polynomials H = {ax + ax? + ax?...

    Let H be the set of third degree polynomials H = {ax + ax? + ax? | aEC} Is H a subspace of P3? Why or why not? Select all correct answer choices (there may be more than one). a. H is a subspace of P3 because it contains the zero vector of P3 b. H is a subspace of P3 because it is closed under vector addition and scalar multiplication Oc H is a subspace of P3 because it...

ADVERTISEMENT
Free Homework Help App
Download From Google Play
Scan Your Homework
to Get Instant Free Answers
Need Online Homework Help?
Ask a Question
Get Answers For Free
Most questions answered within 3 hours.
ADVERTISEMENT
ADVERTISEMENT
ADVERTISEMENT